Gibt es eine Möglichkeit, die Form der Lorentzkraft aus der Relativitätstheorie zu rechtfertigen oder abzuleiten?

Die Lorentzkraft hat diese Form:

F = Q [ E + u × B ]
Wie wir wissen, ist sie Lorentz-invariant. Gibt es eine Möglichkeit, seine Form aus der Relativitätstheorie zu rechtfertigen oder abzuleiten?

Ich glaube nicht, dass die Lorentz-Kraft Lorentz-invariant ist, weil sie von der Geschwindigkeit abhängt. Tatsächlich ist es nicht einmal ein Vierervektor. Können Sie das näher erläutern?
@march seine Form ist unter der Lorentz-Transformation unveränderlich. Ich meine, wenn man sowohl elektrisch als auch magnetisch umwandelt, dann ist die Kraft wieder in dieser Form mit neuen Feldern.
gut, wenn geschrieben F a = Q F a β u β Es scheint ziemlich offensichtlich, dass dies die einzig mögliche kovariante Kraft ist , die linear ist E , B . Aber die Wahrheit ist, nur Experimente können dies rechtfertigen ...

Antworten (2)

Obwohl das Lorentz-Kraftgesetz wie jede andere Physik experimentell gemessen wird, könnte man sich leicht ein alternatives Universum vorstellen, in dem die Entdeckung der Relativitätstheorie vor der elektromagnetischen Theorie stattfand (angenommen, Michelson-Morely lebte vor Faraday, oder schauen Sie sich den Ignatowskischen Ansatz zur speziellen Relativitätstheorie an , wo die Form der Lorentz-Transformation wird ganz ohne Bezug auf Licht argumentiert).

In dieser alternativen Geschichte könnten sich zwei theoretische Physiker unterhalten haben:

Szene 1: A & B sprechen über allgemeine Gesetze zum Thema „Elektrische Ladung“

A: Wir wissen von dieser seltsamen Eigenschaft, die kürzlich entdeckt wurde und die „elektrische Ladung“ genannt wird. Welche Art von Gesetzen würde seine Bewegung regeln?

B: Nun, ich habe gesehen, wie eine Ladung im Labor ziemlich still stand oder sich gleichmäßig bewegte, also ist es eindeutig möglich, dass es keinen elektrischen Effekt gibt. Lassen Sie uns ein elektromagnetisches Feld postulieren F ( X , v , ) ..."

A: Das müssen natürlich alle Vierervektoren sein, oder zumindest etwas, um das Gesetz Lorentz kovariant zu machen ....

B: Natürlich, Dummkopf, ich habe dich nur getestet. Jedenfalls wenn F nach erster Ordnung linear wäre, müßte das Gesetz homogen sein ...

A: .... und Sie könnten loswerden X denn ohne andere Ladungen hängt die Kraft nicht von der Position ab ...

B: Natürlich: Ich habe Sie gerade auch darauf getestet ... also sind wir uns einig, dass es eine lineare, homogene Funktion der Geschwindigkeit ist? ....

A: ... du meinst vier Geschwindigkeiten ...

B: Natürlich: Also ist eine lineare homogene Funktion von vier Geschwindigkeiten das einfachste, plausibelste, was man ausprobieren kann. Hey, hören Sie auf, meine Entdeckung eines neuen Gesetzes zu unterbrechen, würden Sie ...

A: ... unser Gesetz ...

B: Okay. Aber du wirst der zweite Autor sein, okay. Wie auch immer, hier ist, was wir haben: Unser Feld muss ein Lorentz-kovarianter Tensor mit Rang zwei sein, um für eine lineare homogene Form von Newtons zweitem Gesetz zu stehen:

M D v μ D τ = Q F μ v v v

wo wir schreiben Q um die Kopplungsstärke zwischen der "Ladung" und dem Feld zu messen.

A: Vergessen Sie nicht, dass eine Geschwindigkeit von vier eine konstante Einheitsnorm hat .....

B: Oh DUUUH, ich wollte gerade sagen, dass wir aufgrund offensichtlicher Einschränkungen wie z v , v = 1 : also muss die Beschleunigung Minkowski-orthogonal zur Geschwindigkeit sein, das ergibt:

v μ ( η μ σ F σ v + η v σ F σ μ ) v v = 0 ; v η μ σ F σ v + η v σ F σ μ = 0

also die doppelt kovariante Form von F muss schiefsymmetrisch sein, um die Norm der vier Geschwindigkeiten zu erhalten. Sollen wir jetzt veröffentlichen?

A: Ist Ihnen klar, dass dieser vollkommene Klumpen Heaviside der Herausgeber von J. Modern Irreproducible Physics ist ?

B: Also ...?

A: Wir werden das nie in den Druck bringen, es sei denn, wir schreiben alles in seiner völlig beschissenen, kreuzigen, vektoriellen Notation! Er wird sich nie für diesen absoluten Index entscheiden η μ σ F σ v + η v σ F σ μ = 0 , er wird seine Tasse Tee in seinen eigenen Schoß kippen und an seinem klebrigen Brötchen ersticken, sobald er es liest ...

B: Oh, wie machen wir das dann?

A: Nur wenn ich der erste Autor bin .....!

B: Ach so....

Verlassen Sie A & B, um loszugehen und klebrige Brötchen zu essen, während A die Dotty Crossy-Version ableitet

Zweite Szene: Nach Sticky Buns

A: Hier ist die Dotty-Crossy-Version von η μ σ F σ v + η v σ F σ μ = 0 . Es kann genau dann halten, wenn die Kraft .....

B: die Drei -Kraft .....

A: .... ja natürlich: wir reden hier dotty crossy. Wo war ich? Sie kann genau dann gelten, wenn die drei Kräfte wirken gemäß:

F = Q a ( E + v × B )

Wo a ist eine beliebige Skalierungskonstante, die wir in die Ladungsdefinition aufnehmen können, wenn wir wollen, E besteht aus den nichtdiagonalen Elementen der nullten Reihe F Und B besteht aus den drei unabhängigen Elementen der Schiefsymmetrie 3 × 3 unteren rechten Block von F .

(Siehe meine Antwort hier für weitere Informationen).

Die Lorentz-Kraft wird nicht aus der Relativitätstheorie, sondern aus Experimenten begründet. Tatsächlich wird die Lorentz-Kraft verwendet, um die Felder zu definieren E Und B . Sie haben Recht, dass (die Form) Lorentz-invariant ist.

BEARBEITEN: Die meisten Lehrbücher sprechen das Problem an, indem sie sagen, dass die relativistische (4-Vektor-) Form der Lorentz-Kraft die einfachste ist, die gebildet werden kann, und deshalb verwenden wir sie als Arbeitsmodell, ähnlich wie wir den Standardausdruck für den Poynting-Fluss verwenden als Arbeitsmodell, obwohl es viele Möglichkeiten gibt, es zu definieren.

Das einzige andere Argument, das ich kenne (was ich für besser halte), besteht im Wesentlichen darin, zu zeigen, dass die relativistische Form mit der elektrischen Kraft im Ruhesystem eines Teilchens identisch ist und dass daher die Form allgemein in allen Systemen gilt. Die Argumentation geht ungefähr so:

Im System S, in dem ein Teilchen mit der Ladung q (momentan) ruht, ist die Kraft:

F ich = Q E ich

(aus der Definition des elektrischen Feldes E ). Es steht uns frei, einen 4-Vektor der Form zu bilden:

F ich k U k

Wo U k ist die 4-Geschwindigkeit. Im Ruhesystem ist diese 4-Geschwindigkeit:

U k = ( C , 0 )

und so ist es wahr

Q F ich k U k / C = ( 0 , Q E ich )

(Beachten Sie, dass wir durch c geteilt haben, damit die Dinge übereinstimmen). Wir können also sehen, dass die Komponenten von Q F ich k U k / C und die Lorentzkraft im Ruhesystem gleich sind. Und da eine Tensorgleichung, die in einem Rahmen gültig ist, in allen Rahmen gültig sein muss, ist die Form Q F ich k U k / C denn die Kraft muss in allen Rahmen wahr sein.

Ok, aber Sie wissen, was das OP bedeutete, oder? Sie wollen wissen, ob diese Form eine unvermeidliche Folge einer kleinen Reihe „vernünftiger“ Annahmen über die Natur ist. Ja, diese Annahmen müssen durch Experimente verifiziert werden, aber das ist nicht die Frage.
Sie haben Recht, ich habe meine Antwort erweitert, um die übliche Erklärung zu geben :)